Física: Dinámica con 3 bloques #2 | Con Rozamiento

Поделиться
HTML-код
  • Опубликовано: 24 окт 2024

Комментарии • 57

  • @oscarsotocorrales2258
    @oscarsotocorrales2258 4 года назад +12

    Créame que sus videos son un tesoro que aún le falta a muchas personas por encontrar, siempre que busco ayuda para física o matemáticas usted es mi primera opción, la manera en que explica es muy buena y sus ejemplos de verdad son muy buenos. Lo motivo seguir así y verá que en cuestión de tiempo será el canal #1 en esta materia, es cuestión de tiempo. Muchas gracias!

    • @RonnyOnline
      @RonnyOnline  4 года назад +1

      Gracias Oscar por tus palabras, son bien recibidas y créeme que el honor es mío poder colaborar con este contenido, y pues si aún queda mucho trabajo por delante, así que a seguir subiendo contenido útil y buenos ejercicios para todos. Una vez más gracias por el apoyo y la confianza en mi trabajo, que la fuerza siempre te acompañe.

  • @josetonito2193
    @josetonito2193 2 года назад +2

    Muy excelente explicación profesor. Gracias. Que la fuerza nos acompañe.

  • @luiszambrano7380
    @luiszambrano7380 Год назад +1

    Dios lo bendiga, profesor, muy buena explicación.

  • @JesusMartinez-yn2md
    @JesusMartinez-yn2md 3 года назад

    Te felicito Ronny. Excelente explicación, mejor fundamento y análisis, certera acotación lo de los ángulos. Definitivamente no dejas nada al azar.

  • @henryjimenez8390
    @henryjimenez8390 Год назад +1

    Excelente gracias por brindar tus conocimientos por este medio sin duda son de gran ayuda ....

    • @RonnyOnline
      @RonnyOnline  Год назад

      Gracias por tu comentario no olvides que puedes colaborar en el boton de super gracias 🤗

  • @marielamagaliparedesmaidan7665
    @marielamagaliparedesmaidan7665 3 года назад +3

    Me ayudaste en un ejercicio que vengo meses tratando de entender del porque no habia aprobado un final. Gracias.

  • @jose350681
    @jose350681 Год назад

    Mi estimado, excelente sus explicaciones, veo sus videos con mi hijo , así lo ayudo, está en quinto de bachillerato, si en mis tiempos hubiese este tipo de explicaciones y está herramienta más rápido se aprendería, excelente

    • @RonnyOnline
      @RonnyOnline  Год назад

      Gracias por tu comentario no olvides que puedes colaborar en el boton de super gracias 🤗

  • @joaojosedeoliveiraandrade7663
    @joaojosedeoliveiraandrade7663 3 года назад +3

    Ótimo exercício, ótima explicação; muito obrigado professor.

  • @marijojojo7310
    @marijojojo7310 3 года назад +1

    Muchas gracias profee, sus ejercicios me sirvieron muchísimo para mi exámen, explica muy bien :") gracias profe

    • @RonnyOnline
      @RonnyOnline  3 года назад

      Que tal, gracias por tu comentario, que bueno te haya servido, exito

  • @carmensantellan1
    @carmensantellan1 3 года назад +1

    Excelente explicación, muchas gracias

    • @RonnyOnline
      @RonnyOnline  3 года назад

      Gracias por tu comentario, que bueno te haya servido el contenido, bienvenida, suscribete y comparte

  • @lioneljuarez7577
    @lioneljuarez7577 Год назад

    Buenas , creo q deberias explicar lo de sistemas inerciales y no inerciales , ya que en el primer ejercicio , estoy casi seguro q en el bloque de arriba deberia existir una fuerza ficticia que es la q tira el cuerpo hacia atrás.... saludos

  • @ElprofesorA
    @ElprofesorA 3 года назад +1

    Profe ya averigüé con otros profesores y el Sears está correcto. La ff1 en M2 no existe ya que la fricción es la que se opone al desplazamiento y ff1 no se opone al desplazamiento de m2 por tal razón esta fuerza de fricción no debe ir en la sumatoria de fuerzas con respecto a x del bloque m2.

    • @RonnyOnline
      @RonnyOnline  3 года назад

      Cuando hay fricción entre los bloques es una fuerza de tercera ley de Newton y si va en ambos como en este ejemplo:
      ruclips.net/video/o04K4RGOKmw/видео.html
      Todos mis ejercicios son verificados incluso con los solucionarios, pero si tu profesor te dice que no la coloques chevere pero es incorrecto

  • @ElprofesorA
    @ElprofesorA 3 года назад +2

    profe analizando la fuerza de fricción 1 del bloque m2 creería que no existe....Podríamos discutir ese ejemplo después de que mires el solucionario de Sears. Gracias y me parecen muy buenas tus clases exítos!!!

    • @RonnyOnline
      @RonnyOnline  3 года назад

      Solo si el problema dice que no hay fricción entre los bloques entonces no se coloca, pero en este vídeo y en los demás que tengo que hay fricción entre ambos, la ff1 se refleja en la masa2 por tercera ley de Newton está en los textos de física y solucionarios, todos mis vídeos son verificados antes de subirlos por lo delicado que son estos temas

    • @ElprofesorA
      @ElprofesorA 3 года назад

      @@RonnyOnline profe ud me podría compartir algún solucionario de física donde muestren ese ejercicio. Entiendo lo que me dices pero aún me queda la duda. Mira el ejercicio del solucionario de sears 5.88 y veras que no ponen esa fricción

    • @RonnyOnline
      @RonnyOnline  3 года назад +1

      @@ElprofesorA con todo el gusto de mundo, escríbeme al correo: ronnyenlinea@gmail.com

  • @isaiassanchez6629
    @isaiassanchez6629 3 месяца назад

    Puedo tomar a el bloque m1 y m2 como un solo cuerpo? Y de acordé a eso hacer mí diagrama de flujo?

  • @rodrigochaile4862
    @rodrigochaile4862 Год назад

    Disculpe al calcular la tension en la masa 2 no deberiamos hacerlo con la suma de las masas 1 y 2?

  • @eduardovasquez8390
    @eduardovasquez8390 2 года назад +1

    Buena explicacion .......

    • @RonnyOnline
      @RonnyOnline  2 года назад

      Gracias por tu comentario, no olvides suscribirte darle like y comparte

  • @edinsonevervasquezcotrina4078
    @edinsonevervasquezcotrina4078 7 месяцев назад

    Profesor, no entiendo porque en la masa 3 no se ha multiplicado por la gravedad y tampoco lo ha tomado en cuenta en la sumatoria de fy, siendo que esto influye en la normal 3.

  • @jhonatanhernandez8515
    @jhonatanhernandez8515 Год назад

    Profe una pregunta porque en este ejercicio para hallar aceleracion solo lo haces en el primer bloque? no deberia ser la aceleracion de todos?

  • @brandonluissoliz4191
    @brandonluissoliz4191 3 года назад +1

    Muy buena explicacion

    • @RonnyOnline
      @RonnyOnline  3 года назад

      Gracias por tu comentario, que bueno te haya servido el contenido, bienvenido, suscribete y comparte

  • @luissosa8569
    @luissosa8569 2 года назад

    Profe puede explicar porque se aplica el principio de equilibrio. Me confunde porque si el ejercicio es dinámico. Saludos

  • @fallenapophis5568
    @fallenapophis5568 4 года назад +2

    cuando el enunciado dice: m1 y m2 se mueven juntos, nos esta diciendo que la aceleracion de ambos cuerpos es el mismo, no? por que un profe me dijo que no siempre la aceleracion de un cuerpo m3 que esta sobre un cuerpo m4 sean las mismas

    • @RonnyOnline
      @RonnyOnline  4 года назад +1

      Lo que dice el profesor es correcto, pero el problema siempre lo va a especificar, acá te comparto ese caso:
      ruclips.net/video/o04K4RGOKmw/видео.html

  • @neftalydejesussuchiteabzun5578
    @neftalydejesussuchiteabzun5578 3 года назад +1

    Excelente

    • @RonnyOnline
      @RonnyOnline  3 года назад

      Gracias por tu comentario, que bueno te haya servido el contenido, bienvenido, suscribete y comparte

  • @agathanavarronaranjo9898
    @agathanavarronaranjo9898 3 года назад

    excelente video, muchas gracias pero el peso 2 del cuerpo dos no debería ser la suma de las masas del cuerpo uno y el dos por g?

  • @Francisco_avalos_
    @Francisco_avalos_ 2 месяца назад

    Gracias a usted aprobaré mi examen de Admisión profe, espero no deje de hacer videos ya que lo enseñas muy bien.

  • @Rodrigo-n4r
    @Rodrigo-n4r Месяц назад

    La masa 1 y la masa 2 se mueven juntas, porque no lo toma como sistema y asi su masa (del sistema) seria 8 por la aclaración

  • @gustavoesparzasierra5750
    @gustavoesparzasierra5750 2 года назад

    no entiendo la de la friccion de la masa 1 no la entiendo, creo que debe ser negativa al sentido contrario de la direccion que va a desplazarse junto a la masa 2

  • @melanypitti5801
    @melanypitti5801 3 года назад

    porque la normal 1 es negativa?

  • @bladymolina3342
    @bladymolina3342 4 года назад +1

    muy bueno

    • @RonnyOnline
      @RonnyOnline  4 года назад

      Gracias Blady por este comentario, un placer subir este contenido. Bienvenidos a mi canal

  • @Prof.Plinio_Teheran
    @Prof.Plinio_Teheran 2 года назад

    Ejercicio bien realizado con procedimiento aritmético-algebraico, sin embargo algunas afirmaciones en sus comentarios(al margen) de la explicación sería aconsejable que se editaran, son imprecisiones conceptuales que inducen a errores comunes en el estudiantado.

  • @anabelecheverria3401
    @anabelecheverria3401 Год назад

    Cuando multiplico 15x2.94 sería la fuerza motriz

  • @leofabianferreyralopez1626
    @leofabianferreyralopez1626 4 года назад +1

    porque calculo la Fuerza de fricción 1 como la Normal 1 x el Coeficiente de roce dinámico? se supone que la fricción es estática ya que los cuerpos A y B se mueven juntos. Lo que hizo solo seria valido si supiera que los cuerpos tienen aceleraciones distintas.

    • @RonnyOnline
      @RonnyOnline  4 года назад

      Que tal Leo, el bloque 1 y el 2 se mueven juntos por tanto la fricción entre ellos es estática y la calculo con 0.3 que es el coeficiente estático
      Luego la masa 2 se mueve sobre el piso y ahí se coloca el coeficiente dinámico que es el que pide el problema
      El caso que comentas de bloques con aceleraciones diferentes acá te lo dejo:
      ruclips.net/video/o04K4RGOKmw/видео.html

    • @leofabianferreyralopez1626
      @leofabianferreyralopez1626 4 года назад

      @@RonnyOnline disculpe, me equivoque y puse coeficiente dinámico en vez de estático. Pero a lo que voy es que la Fuerza de fricción 1 = Normal 1 x el Coeficiente de roce estático sii estamos hablando de la Fuerza de fricción estática máxima. En este problema solo sabemos que Ffr1

    • @RonnyOnline
      @RonnyOnline  4 года назад

      @@leofabianferreyralopez1626 totalmente de acuerdo pero como también puede ser el valor Máximo, lo que sucede en estos problemas es que ya te dicen que se mueven juntos y te dan el valor a usar directamente
      No te preguntan si se mueven juntos y la verificación primero
      Pero si, tú observación es correcta

    • @leofabianferreyralopez1626
      @leofabianferreyralopez1626 4 года назад +1

      @@RonnyOnline ahh claro claro, de echo seria imposible resolver el ejercicio sin usar así ese dato ya que tendríamos 4 incógnitas y 3 ecuaciones, gracias por responder.

    • @RonnyOnline
      @RonnyOnline  4 года назад

      @@leofabianferreyralopez1626 un gusto poder comentar acá y Bienvenido a mi canal

  • @gamermaxbogotaroyal4669
    @gamermaxbogotaroyal4669 3 года назад +1

    Profe la fuerza neta en el bloque 1 deberia tener en cuenta las masas m1 y m2 es decir la aceleración depende del peso de las dos masas y ud puso solo m1a

    • @RonnyOnline
      @RonnyOnline  3 года назад +1

      Hola que tal, cuando se hacen los diagramas de cuerpo libre por masas individuales solo se colocan las fuerzas que tiene está, y efectivamente la aceleración de la masa 1 y 2 es la misma por eso en los cálculos como puedes ver se calcula con la masa 1 y se usa esa misma en la 2, el método que ves está en los textos recomendaciones como el SERWAY

    • @RonnyOnline
      @RonnyOnline  3 года назад

      P.D.: en el video puse la m2 como referencia al hacer el diagrama en la m1 no es que se debe sumar esa masa

  • @josedanielhanccovargas1705
    @josedanielhanccovargas1705 3 года назад

    mi calculadora y mi cerebro se contradicen
    xd q hago

  • @juaniribarren4550
    @juaniribarren4550 4 месяца назад

    Podría hablar un poco más rápido